Presentation is loading. Please wait.

Presentation is loading. Please wait.

4th Period Review Projects

Similar presentations


Presentation on theme: "4th Period Review Projects"— Presentation transcript:

1 4th Period Review Projects
2015

2 LO 3.19 The student is able to describe the connection between the regulation of gene expression and observed differences between individuals in a population. SP 7.1 The student can connect phenomena and models across spatial and temporal scales. Explanation: In the process of histone acetylation, for example, acetyl groups are added onto histone tails, the amino acids protruding from nucleosomes, which DNA wraps around to form chromatin. This acts as an indirect promoting agent for transcription because acetylation loosens chromatin structure by neutralizing the charge of histone tails, weakening the bonds between nucleosomes (Fig. 1.1). This causes them to separate, exposing DNA to transcription factors. The following transcription and translation of these once-unreachable sequences gives rise to new proteins that open signal transduction pathways in cells, causing new cellular responses. For example, some allergies are caused by histone acetylation that exposes genes within the nuclei of immunological T-cells that code for certain membrane-bound protein receptors. The new receptors cause an allergic reaction in response to a specific antigen. M.C. Question Which of the following is true concerning gene expression? a) DNA sequence is the only determinant of heritable traits b) Any cell can transcribe any gene in its genome c) Adding methyl groups to DNA bases blocks transcription of genes d) RNA strands are degraded by ribosomes after use Learning Log/FRQ-Style Question Gene expression not only involves genes in DNA form, but also proteins that they code for. Describe in detail how genes are regulated via protein degradation. Cancerous tumors are caused by loss of tumor suppressor genes. Explain the function of tumor suppressor proteins and how cancer is caused by their absence. Figure 1.1

3 Answer Key – LO 3.19 Which of the following is true concerning gene expression? a) DNA sequence is the only determinant of heritable traits b) Any cell can transcribe any gene in its genome c) Adding methyl groups to DNA bases blocks transcription of genes d) RNA strands are degraded by ribosomes after use Gene expression not only involves genes in DNA form, but also proteins that they code for. Describe in detail how genes are regulated via protein degradation. Cancerous tumors are caused by loss of tumor suppressor genes. Explain the function of tumor suppressor proteins and how cancer is caused by their absence. After a protein has been used in a cell, it is degraded by ubiquitin and proteasomes. The molecule ubiquitin binds to the protein to be degraded and tags it. The ubiquitin signals a large protein complex called a proteasome that wraps around the tagged protein and encloses it. The proteasome then unfolds the protein and its enzymatic properties cut the protein into smaller, more manageable peptides that are later degraded by smaller enzyme activity in the cytosol. This way, the product of the expressed gene is eliminated, thus regulating its expression. The normal function of a tumor suppressor gene is to code for a protein that repairs damaged DNA in order to prevent the spread of mutations in a cell, which helps prevent uncontrolled cell growth and cancerous mutations. When the gene is lost or inactive, the tumor suppressor proteins cannot be synthesized; therefore, any mutations that occur in the DNA that are normally screened by tumor suppressor proteins may be able to slide by and evade repair. If the mutation is disruptive, it could interrupt the growth pattern of the cell and cause growth of a tumor, which will continue to grow uncontrollably by nature and lead to cancer.

4 Learning Objective 4.23: The student is able to construct explanations of the influence of environmental factors on the phenotype of the organism. Science Practice 6.2: The student can construct explanations of phenomena based on evidence produced through scientific practices. Explanation: A phenotype is the physical and psychological traits of an organism, which is determined by its genetic makeup. Some environmental factors have negative and positive effects on the phenotype of an organism. A common example of the effects of the environment on the phenotype of an organism are hydrangea flowers of the same genetic variety. Due to the acidity of the soil they are planted in, the flowers can range anywhere in color from blue-violet to pink. Multiple Choice Question: Based on your knowledge of environmental phenotypic effects, which of the following is incorrect? The environmental effects on the phenotype can evolutionarily benefit an organism Organisms are pre disposed to what environmental factors effects their phenotype due to their size Identical twins, that are genetically the same, gain phenotypic differences as a result of their different experiences The phenotypic range are phenotypic possibilities due to environmental influence Learning log/FRQ style: Suppose identical twins who were given up for adoption to two different families found each other later in life and look nothing alike. Twin A grew up in a wealthy home with parents that exposed it to many different types of foods and many different life experiences. Twin B grew up in multiple foster homes, did not always receive meals, and was surrounded by many illegal things. Explain factors of their environments that could have attributed to their phenotypic differences.

5 Answer key learning objective 4.23
Multiple Choice Question: Based on your knowledge of environmental phenotypic effects, which of the following is incorrect? The environmental effects on the phenotype can evolutionarily benefit an organism Organisms are pre disposed to what environmental factors effects their phenotype due to their size Identical twins, that are genetically the same, gain phenotypic differences as a result of their different experiences The phenotypic range are phenotypic possibilities due to environmental influence Learning log/FRQ style: Suppose identical twins who were given up for adoption to two different families found each other later in life and look nothing alike. Twin A grew up in a wealthy home with parents that exposed it to many different types of foods and many different life experiences. Twin B grew up in multiple foster homes, did not always receive meals, and was surrounded by many illegal things. Explain factors of their environments that could have attributed to their phenotypic differences. Considering the fact that they are identical twins, and that they have the exact same genetic makeup it would be expected for them to look exactly the same. Twin A had access to healthier foods at a young age which stimulated growth causing a beneficial effect. Twin B missed meals which can stunt growth which would explain a difference in height. Due to twin B being around illegal substances such as drugs or even cigarettes at a young age can account for early signs of secondhand smoke which can cause physical changes in a person.

6 LO 3.21 The student can use representations to describe how gene regulation influences cell products and function. SP 1.4 The student can use representations and models to analyze situations or solve problems qualitatively and quantitatively. Explanation: All organisms must regulate which genes are expressed at any given time. Both unicellular organisms and the cells of multicellular organisms must continually turn genes on and off in response to signs from their external and internal environments. The cells of a multicellular organism must also regulate their gene expression on a more long-term basis. During development of a multicellular organism, its cells undergo a process of specialization in form and function. Eukaryotes have a great variety of control mechanisms operating before transcription and after translation (Figure 1.1). An example of gene expression is an individual E. coli cell living in a human colon, dependent for its nutrients on its host. If the environment is lacking tryptophan (amino acid), then they cell responds by starting a metabolic pathway that makes tryptophan from another compound. If the host later eats a tryptophan-rich meal, the cell stops producing its own tryptophan. Cells can adjust the amount being made of certain enzymes - meaning they can regulate the expression of genes encoding the enzymes (Figure 1.2). This control of enzyme production occurs at the transcription level. Many genes of the bacterial genome are switched on or off by the control of an operon. Inducers turn on genes, while repressors inhibit them. Genes such as ribosomal genes are always expressed, or turned on. MC Question: Which of the following accurately represents an active repressor and an operon turned on in the absence of lactose? FRQ Question: Go over and analyze Figure 1.2. On what two levels does metabolic control occur? Explain the length of response. What do the red symbols signify? Figure 1.1 a) c) b) d) Figure 1.2

7 The correct answer is a. LO 3.21 Answer Key a) c) b) d)
MC Question: Which of the following accurately represents an active repressor and an operon turned on in the absence of lactose? a) c) The correct answer is a. b) d) FRQ Question: Go over and analyze Figure 1.2. On what two levels does metabolic control occur? Explain the length of response. What do the red symbols signify? An abundance of tryptophan can inhibit the activity of the first enzyme in the pathway, which is feedback inhibition. That is a rapid response. Cells can adjust fairly quickly to already present cells. The response relies on the sensitivity of many enzymes to chemical cues that can increase or decrease their catalytic activity. It can also repress expression of the genes for all the enzymes needed for the pathway, which is a longer term response. Repressors turn off operons, while inducers turn them on. The red circles signify inhibition.

8 LO 4.5: The student is able to construct explanations based on scientific evidence as to how interactions of subcellular structures provide essential functions SP 6.2: The student can construct explanations of phenomena based on evidence produced through the scientific process. Explanation: Ribosomes become the site of protein synthesis where translation yields necessary polypeptides for the cell. The Endoplasmic reticulum (ER) has two types: smooth and rough. The smooth ER synthesizes lipids. The rough ER supports and compartmentalizes the cell, helps in intracellular transport of molecules to other organelles and provides protein synthesis with the ribosomes embedded in it. The Golgi complex produces lysosomes and packages molecules for transport in vesicles, which move molecules between organelles. Mitochondria provide energy in the form of ATP that is sent to power the functions of the other cells. Lysosomes digest, and recycle organic materials (such as old broken organelles) and help in apoptosis. A vacuole helps digest and get rid of cellular waste, which other organelles create. Chloroplasts (only found in plants and algae) also provide energy. M.C. Question: Which of the following is least important in the production of proteins? Ribosomes The Endoplasmic reticulum The Golgi complex Lysosomes FRQ-style Questions: Jerry just found out that his cells are not producing enough insulin, a hormone signal that is proteins secreted by the cell. What cellular structures are involved in the creation and secretion of proteins and what are their roles?

9 The Endoplasmic reticulum The Golgi complex Lysosomes
M.C. Question: Which of the following is least important in the production of proteins? Ribosomes The Endoplasmic reticulum The Golgi complex Lysosomes FRQ-style Questions: Jerry just found out that his cells are not producing enough insulin, a hormone signal that is proteins secreted by the cell. What cellular structures are involved in the creation and secretion of proteins and what are their roles? Proteins will be created by ribosomes, some of which are free floating in the cytosol, and some of which are found in the rough endoplasmic reticulum. The proteins will be taken by vesicles to the golgi apparatus, where the proteins will be manufactured and polished fully, before being shipped to the plasma membrane, where they are secreted out of the cell. Figures from: Pearson Education Inc

10 L.O. 1.32: The student is able to justify the selection of geological, physical, and chemical data that reveal early Earth conditions. S.P 4.1: The student can justify the selection of the kind of data needed to prove the early Earth conditions using experimentation and reasoning. Explanation: Geological evidence involving the origin of life on Earth includes facts that the Earth was formed approximately 4.6 billion years ago, but the environment was too hostile for life until 3.9 billion years ago. The earliest fossil shows signs that prove life as early as 3.5 billion years ago. Looking at this evidence combined, the dates create a time period when the origin of life could have occurred. Multiple chemical experiments have also shown that it is possible to form complex organic molecules from inorganic molecules. These experiments explain background regarding the building blocks of life, changing from a monomer, to a polymer, creating a membrane, and then finally a cell. There is also evidence showing that all organisms on Earth share a common ancestral origin of life. This includes a common genetic code and similar molecular building blocks found in all life forms. M.C Question: Which of the following statements regarding the evolution from monomers to polymers is FALSE? Newly formed monomers polymerized to produce larger molecules, polymers Heat had an impact on the vaporization of water and linked the monomers into polymers The high levels of O2 in the atmosphere created bonds to create polymers from monomers Monomers accumulated in the ocean, an environment suitable for the production and multiplication of polymers Learning Log/FRQ-style Question: Explain, in detail, the physical conditions of primitive Earth. Also, describe why RNA had to be the first genetic material.

11 ANSWER KEY – L.O. 1.32 M.C Question: Which of the following statements regarding the evolution from monomers to polymers is FALSE? Newly formed monomers polymerized to produce larger molecules, polymers Heat had an impact on the vaporization of water and linked the monomers into polymers The high levels of O2 created bonds to create polymers from monomers Monomers accumulated in the ocean, an environment suitable for the production and multiplication of polymers Primitive Earth had no aerobic organisms for cellular respiration to occur, so levels of O2 were low. Also, if there was O2 during this time, oxidation could have occurred and monomers might not have evolved into polymers in the same amount of time or not at all. All of the other statements are true. Explain, in detail, the physical conditions of primitive Earth. Also, describe why RNA had to be the first genetic material. Primitive Earth’s atmosphere consisted of little amounts of O2 (low amounts of respiration levels) and an abundance of H2O, CH4, CO, CO2, and N2. The environment was volatile with lightening and frequent volcanic activity. This activity created an Earth that was constantly being heated up and cooled down. Because there was little amounts of O2 in the atmosphere, there was also no ozone layer. The lack of an ozone layer increased the levels of UV radiation from the sun. Primitive Earth was also constantly being bombarded with meteorites. RNA was the first genetic material because of it’s simplicity. RNA is single stranded and is replicated by ribozyme, a RNA molecule. Collections of these simple RNA molecules that are best suited for the environment replicate their RNA and reproduce. Another factor of why RNA was the first genetic material is the interactions between mRNA, tRNA, and tRNA during translation. These interactions created the building block for life. As shown in Figure 1.1, the production of RNA created a chain of events leading to DNA strands now. Without the origin of RNA, the process would be incomplete creating a gap in the theory of origin. Figure 1.1

12 LO 4.13: The student is able to predict the effects of a change in the community’s populations on the community. SP 6.4: The student can make claims and predictions about natural phenomenas based on scientific theories and models. Explanation: A way that this LO can be incorporated into curriculum is when the ecology unit of Biology is being taught, because graphs and other visual data are displayed about communities and populations most frequently during this unit. With these graphs, the student can find different items such as carrying capacity, the j-curve, etc. and will be able to extract data from such patterns. The LO is connected to the SP because while the student is observing the graph and finding patterns, the student is then able to make predictions about the population from the patterns and data that is shown. Also, theories that apply to the model, the student will be able to extract information from the model and connect it to different scientific theories. M/C Question: According to this graph, which of the following would be true if hyenas were to enter the population? The hyena is a known predator of the baboon. a.) The baboon population will decrease due to increased predation and the cheetah population will increase due to a decrease in competition. b.) The baboon and cheetah population will remain the same due to the large population of the baboon. c.) The baboon population would increase due to competition between the predators while the cheetah population will decrease due to increase competition. d.) The baboon population would decrease due to increased predation and the cheetah population would decrease due to an increase in competition for food. FRQ-style question: (a.) Based on the graph above, describe three factors that could limit the baboon population between years 8 and 12 and explain how those factors could have directly affected the baboon and cheetah population. (b.) Draw the survivorship curve for both k-selected and r-selected species, give one example for each, and explain how each examples affects the other.

13 Answer Key—LO 4.13 M/C Question: According to this graph, which of the following would be true if hyenas were to enter the population? a.) The baboon population will decrease due to increased predation and the cheetah population will increase due to a decrease in competition. b.) The baboon and cheetah population will remain the same due to the large population of the baboon. c.) The baboon population would increase due to competition between the predators while the cheetah population will decrease due to increase competition. d.) The baboon population would decrease due to increased predation and the cheetah population would decrease due to an increase in competition for food. FRQ: (a.) Based on the graph above, describe three factors that could limit the baboon population between years 8 and 12 and explain how those factors could have directly affected the baboon and cheetah population. (b.) Draw the survivorship curve for both k-selected and r-selected species, give one example for each, and explain how each group affects the other. FRQ Answer: (a.) Since the time period that is being focused on is during a rapid increase in the baboon population, the main factors that would risk the baboon population would be density-dependent factors. Three density-dependent factors include resource limitation, health, and predation. Resource limitation would be a large problem to the baboons because the environment may not be able to produce necessary resourced needed for the baboon with such a rapid increase in the population. Health is a factor because a large population of baboons in a small area can spread disease easier and thus kill off a portion of the population. Predation is a factor because more baboons can spread out within an area and lure predators ,other than the cheetah, to the baboon niche for easy food supply. The cheetah population would decline due to increase predation. (b.) An example of a k-selected species are humans because they survive for a longer time and have a lower death rate as well as have fewer offspring. An example of an r-selected species is a honeybee because the lifespan is shorter and the death rate is higher, but have many more offspring. The honeybee has the ability to pollinate flowers and make honey, which humans use the pollinated flowers (which produce fruits) and honey as a food source. If the honeybee were to die out, a large percentage of human food would be gone as well. If the human population was to die out, then the bees would not have a reason to pollinate the flowers and would die out.

14 LO 2.1 : The student is able to explain how biological systems use free energy based on empirical data that all organisms require constant energy input to maintain organization, to grow, and to reproduce. SP 6.2:The student can construct explanations of phenomena based on evidence produced through scientific practices Explanation: Organization: Constant free energy must be flow into the system in order to maintain order otherwise the lack of free energy and order results in death. Entropy(disorganization), the source in a lack of order, increases over time. An example of entropy can be found in trophic levels (Fig 1) where 90% of energy is lost as one moves up the diagram. Power coupling the rxns with negative changes in free energy (increase entropy) with rxns resulting in positive free energy changes (decrease entropy) are necessary to ensure the energy input exceeds the free energy lost to entropy in order to ensure the organism can carry out the metabolic functions necessary for survival. The conversion of ATP ADP is an exergonic rxn that increases entropy; however, it is necessary for metabolic functions so positive free energy changes are paired with it to ensure the organism can survive. Growth: When excess free energy is accumulated, it is stored or used for growth because it is not needed for the required free energy that aids in the necessary metabolic functions for survival. This means that GPP (Gross Primary Productivity-the rate producers in an ecosystem capture and store a given amount of chemical energy as biomass in a given span of time) and NPP (Net Primary Productivity-the measurement rate of primary producers in an ecosystem to produce net useful chemical energy) also play a primary role in growth due to producers being an intrinsic role to trophic levels and the amount of free energy available. Reproduction: Various organisms have adapted to expected fluctuations in free energy availability by developing various reproductive strategies. An example of this is in mating patterns such as seasonal reproduction aid in ensuring that there is substantial resources to where they are able to successfully mate and rear viable offspring. Reproductive diapause is a phenomena that growth is put to a halt or slowed down in response to conditions being unfavorable thus resulting in a lack of free energy; thus, it explains why populations fluctuate according to the amount of free energy available because if there isn’t enough around, then either organisms die or less offspring survive. MC Question: The process of converting ATP into ADP, a part of cellular respiration, is an exergonic reaction that results in a loss of free energy. In order to maintain organization, what must be done to maintain organization? Nothing needs to be paired because the loss of free energy means a decrease in entropy. ADP to ATP is an exergonic rxn that will decrease entropy. ADP to ATP is an endergonic rxn that will decrease entropy. NADPH to NAD is exergonic and will increase entropy. FRQ: A scientist observing an ecosystem in the prairies has been recording the energy flow between the trophic levels. The vegetation in the past had produced 10,500 kcal of energy; however, a resent brush fire affected this year’s GPP thus, only 9,000 kcal was produced. The organisms that follow are the prairie dogs, rattle snakes, and hawk. make a diagram of the trophic levels with the kcals of before and after the brushfire. Label it accordingly. Identify and explain a result from this change that would affect the organisms?

15 Answer Key- LO 2.1 a. Prairie Trophic Levels
The process of converting ATP into ADP, a part of cellular respiration, is an exergonic reaction that results in a loss of free energy. In order to maintain organization, what must be done to maintain organization? Nothing needs to be paired because the loss of free energy means a decrease in entropy. ADP to ATP is an exergonic rxn that will decrease entropy. ADP to ATP is an endergonic rxn that will decrease entropy. NADPH to NAD is exergonic and will increase entropy. A scientist observing an ecosystem in the prairies has been recording the energy flow between the trophic levels. The vegetation in the past had produced 10,500 kcal of energy; however, a recent brush fire affected this year’s GPP thus, only 9,000 kcal was produced. The organisms that follow the primary producers are the prairie dog, rattle snake, and hawk. make a diagram of the trophic levels with the kcals of before and after the brushfire. Label it accordingly. Identify and explain a result from this change that would affect the organisms? a. Prairie Trophic Levels Before After Tertiary Consumer: Hawk kcal kcal b. Due to a decrease in free energy available, it will be more difficult for organisms to obtain the amount of energy needed to carry out cellular respiration efficiently enough to where the populations of these organisms will decrease from either high death rates or a reduction in offspring produced that will survive during this environmental pressure. Secondary Consumer: Rattle Snake kcal kcal Primary Consumer: Prairie Dogs kcal kcal Primary Producer: Grass ,500kcal ,000kcal

16 LO 1.12 The student is able to connect scientific evidence from many scientific disciplines to support the modern concept of evolution [See SP 7.1] SP 7.1 The student can connect phenomena and models across spatial and temporal scales Explanation: Scientific evidence of biological evolution uses information from geographical, geological, physical, chemical and mathematical applications. Students should be able to connect evidences like cladograms, phylogenetic trees, carbon dating, allele frequencies, etc. in order to back the theory of evolution. MCQ: Which of the following is not a direct cause of microevolution? Genetic Drift Selective mating Polymorphisms Gene flow FRQ: Phylogeny is a way to depict the evolutionary history of a species. Given this table, create a cladogram based on their evolutionary developments. Identify and explain two mechanisms of population change. Jaws Lungs Claws/Nails Feathers Fur Hagfish - Perch + Salamander Lizard Pigeon Mouse Chimp

17 ANSWER KEY- LO 1.12 Chimp Mouse Pigeon Lizard Salamander Perch Fur
Which of the following is not a direct cause of microevolution? A. Genetic Drift B. Selective mating C. Polymorphisms D. Gene flow FRQ: Phylogeny is a way to depict the evolutionary history of a species. Given this table, create a cladogram based on their evolutionary developments. Identify and explain two mechanisms of population change. Two examples of population change are through the bottleneck effect and gene flow. In the bottleneck effect, the population is changed via the gain or loss of alleles due to immigration/emigration. In the bottleneck effect, the population size is decreased dramatically due to a disaster and the new population is not representative of the original population, facilitated by natural selection. Chimp Mouse Pigeon Lizard Salamander Perch Hagfish Fur Feathers Claws/Nails Lungs Jaws

18 LO 3.14: The student is able to apply mathematical routines to determine Mendelian patterns of inheritance by data sets. SP 2.2: The student can apply mathematical routines to quantities that describe the natural phenomena. Explanation:The inheritance patterns of single gene diseases are often referred to as Mendelian since Gregor Mendel first observed the different patterns of gene segregation for selected traits in garden peas and was able to determine probabilities of recurrence of a trait for subsequent generations. For example Mendel discovered that, when he crossed purebred white flower and purple flower pea plants (the parental or P generation), the result was not a blend. Rather than being a mix of the two, the offspring (known as the F1 generation) was purple-flowered. When Mendel self-fertilized the F1 generation pea plants, he obtained a purple flower to white flower ratio in the F2 generation of 3 to 1. And you can see the results by using a punnett square. MC Question: A South American lizard has two possible gene traits for skin color: Green which is dominant or Blue which is recessive. Assume two of these lizards mate; one with the dominant allele and one with the recessive allele. If you knew the dominant allele was heterozygous, what would be the outcome of the F1 generation with four offspring? 100% green, 0% blue 75% green, 25% blue 50% green, 50% blue 25% green, 75% blue Learning Log/FRQ-style questions: On an island in the south pacific a new species of fly was discovered. Several crosses made between 50 male flies and 50 female flies. All the F1 offspring had normal wings. F1 flies were crossed, and the data for the resulting F2 flies are given. The results were for males: 365 with stunted wings and 890 with normal wings. And for females 300 stunted and 850 with normal. What conclusions can be drawn from cross 1? Explain your answer with support from the data given.

19 Answer Key- LO 3.14 A South American lizard has two possible gene traits for skin color: Green which is dominant or Blue which is recessive. Assume two of these lizards mate; one with the dominant allele and one with the recessive allele. If you knew the dominant allele was heterozygous, what would be the outcome of the F1 generation with four offspring 100% green, 0% blue 75% green, 25% blue 50% green, 50% blue 25% green, 75% blue Explanation: Knowing that the dominant allele of the parent generation is heterozygous and the recessive allele, obviously being homozygous, you can use a punnett square to help solve the problem. Punnett Squares are the simplest form of using Mendels pattern of inheritance. Learning Log/FRQ-style Question: On an island in the south pacific a new species of fly was discovered. Several crosses made between 50 male flies and 50 female flies. All the F1 offspring had normal wings. F1 flies were crossed, and the data for the resulting F2 flies are given. The results were for males: 365 with stunted wings and 890 with normal wings. And for females 300 stunted and 850 with normal. What conclusions can be drawn from cross 1? Explain your answer with support from the data given. From the data given the normal wings are the dominant allele and all the heterozygous alleles express the dominant trait. And the F2 generation shows a 3:1, normal wings to stunted wings. Also there is a correlation for the alleles for both genders. The male and females both have about the same number of individuals with the trait. Also the gene is autosomal and not sex-linked since both of the gneders have about the same amount of individuals with the trait.

20 Free Response Question
LO 3.27: The student is able to compare and contrast processes by which genetic variation is produced and maintained in organisms from multiple domains SP 7.2:The student can connect concepts in and across domain(s) to generalize or extrapolate in and/or across enduring understandings and/or big ideas Genetic variation in prokaryotes is generally achieved through transformation, via taking in naked DNA, transduction, via viral transmission of genetic variation, conjugation, via cell-to-cell transfer, and transposition, via movement of DNA segments within and between DNA molecules. Genetic variation in eukaryotes is increased through sexual reproduction and the possibilities in fertilization. Specifically in sexual reproduction, crossing-over between chromosomes during meiosis (see figure to the left) and random assortment, different chromosomes found in every mate, increase variation. Both prokaryotes and eukaryotes receive increased genetic variation because of the imperfect nature of DNA replication and DNA repair. Which of the following contributes to genetic variation in prokaryotes but not eukaryotes? Different combinations of chromosomes are shared because of the varying DNA among mates. Errors occurring during the process of old DNA strands being replicated into new DNA strands. The crossing over between homologous chromosomes during meiosis. Mistakes that happen during the reparation of damaged DNA The movement of DNA segments between DNA molecules during transposition Free Response Question Identify and describe three differences of how genetic variation is achieved separately in prokaryotes and eukaryotes. Describe how evolutionary advantages are gained through genetic variation. Give a specific example of this.

21 Free Response Question
The correct answer is E, the movement of DNA segments between DNA molecules during transposition. Transposition is the only process provided that is specific to prokaryotes, the other answer choices describe either sexual reproduction or other processes only existent in eukaryotes. Which of the following contributes to genetic variation in prokaryotes but not eukaryotes? Different combinations of chromosomes are shared because of the varying DNA among mates. Errors occurring during the process of old DNA strands being replicated into new DNA strands. The crossing over between homologous chromosomes during meiosis. Mistakes that happen during the reparation of damaged DNA The movement of DNA segments between DNA molecules during transposition Free Response Question Identify and describe two differences of how genetic variation is achieved separately in prokaryotes and eukaryotes. Describe how evolutionary advantages are gained through genetic variation. Give a specific example of this. Crossing over of genes across homologous chromosomes is a phenomenon that increases genetic variation that is present in eukaryotes, but not prokaryotes. During transposition, DNA segments move within DNA molecules, which increases genetic variation. Transposition occurs in prokaryotes but not eukaryotes. Genetic variations give organisms the ability to react to environmental pressures, and over time are the reason organisms are able to evolve. The separate DNA ensure that at some point there will be a single organism within a species that can survive and/or reproduce better than others, thus starting natural selection, which in turn drives evolution. If the Ink Blot Lemur’s (made up, obv.) diet depended on snatching a insect out of trees with their claws and that insect started to live deeper in the tree, genetic variation allows for the possibility of a new lemur with longer claws, better suited to getting the deeper dwelling insects, and that lemur would survive and reproduce. Genetic variation gave the lemur a evolutionary advantage.

22 LO 4.15: The student is able to use visual representations to analyze situations or solve problems qualitatively to illustrate how interactions among living systems and with their environment result in the movement of matter and energy. SP 1.4: The student can use representations and models to analyze situations or solve problems qualitatively or quantitatively. Explanation: Living organisms interact with themselves and their environment in various cycles including the nutrient cycle, nitrogen cycle, energy cycles, water cycle, and many more. Many of these cycles move physical matter, such as chemicals or macromolecules necessary for life; or energy, such as heat or light energy passing to the environment, or chemical energy in the form of glucose or starches that the organism will consume and break down into ATP. MC Question: In the model to the right, which BEST describes the role of the plant? a) To create a suitable environment for other organisms. b) To turn solar energy to chemical for nutrients for the animal. c) To aid in fixation of available nutrients. d) To prevent the loss erosion of available nutrients. Learning Log/ FRQ-style question: Draw a pyramid diagram of the energy passed between trophic levels of a food chain, going from primary producer to tertiary consumer. Using this diagram, explain how energy is lost between trophic levels. Which organism will be most affected by a toxin released into the environment and why?

23 MC Question: In the model to the right, which BEST describes the role of the plant? a) To create a suitable environment for other organisms. The surrounding environment is based off many factors, both biotic and abiotic, and not the sole responsibility of the plants; and does not factor into the nutrient cycle diagram presented. b) To turn solar energy to chemical for nutrients for the animal. The plant photosynthesizes light energy into chemical nutrients that the animal can consume and can be passed through the system. c) To aid in fixation of available nutrients. Plants do not to this, according to the diagram given d) To prevent the erosion of available nutrients. While plants with root systems aid in erosion prevention, it is not a major factor in this cycle. Learning Log/ FRQ-style question: Draw a pyramid diagram of the energy passed between trophic levels of a food chain, going from primary producer to tertiary consumer. Using this diagram, explain how energy is lost between trophic levels. Which organism will be most affected by a toxin released into the environment and why? (See example diagram on the right) Only about 10% of the energy from one trophic level passes on to the next level due to heat loss to the environment and the use of the energy by the organism. Because of energy lost between trophic levels higher level predators are required to consume more of the lower levels, resulting in a buildup of toxins that may not affect the lower levels, making the higher level consumers, such as the third- level consumer in this diagram, much more susceptible to toxins released in the environment. Photos from shawmst.org ( ) cnrit.tamu.edu ( )

24 LO 1.18: The student is able to evaluate evidence provided by a data set in conjunction with a phylogenetic tree or a simple cladogram to determine evolutionary history and speciation. SP 5.3: The student can evaluate the evidence provided by data sets in relation to a particular scientific question. Explanation: Phylogenetic trees and cladograms are models of evolution, and they can be tested. They represent traits that are either lost or gained during the evolutionary process. The evolutionary process describes the way in which species have progressed and split from a common ancestor. Phylogenetic trees and cladograms also portray speciation and how closely organisms share a common ancestor. There are two main types of speciation: allopatric and sympatric. Allopatric speciation occurs when a geographic barrier isolates a population (blocks gene flow). Sympatric speciation occurs when intrinsic factors such as chromosomal changes or non-random mating alter gene flow. Cladograms and phylogenetic trees can be constructed from (but not limited to) homologous structure similarities or similarities in protein/DNA sequences. Phylogenetic trees and cladograms are also never set in stone. They change as we expand our knowledge of evolution. A student can determine how closely related two organisms are by following a cladogram. For example, given Figure 1, the student can determine that a tuna is more closely related to the salamander than it is to the turtle. Table 1: Figure 1: Species Four limbs Amniotic egg Eggs with shells European Tree Frog + - American Crocodile American Blue Rabbit MC Question: Given Table 1 above, correctly give the order of each species on a cladogram that is read from left to right. American Crocodile, American Blue Rabbit, European Tree Frog American Blue Rabbit, European Tree Frog, American Crocodile European Tree Frog, American Blue Rabbit, American Crocodile American Crocodile, European Tree Frog, American Blue Rabbit Table 2: Species Lungs Claws or Nails Feathers Fur; mammary glands A + - B C D E FRQ: Given Table 2, construct a cladogram including all of these species. Then, explain what it means in terms of the evolutionary process and speciation. Provide examples of possible species for species A-E.

25 Table 1: Links: Species Four limbs Amniotic egg Eggs with shells European Tree Frog + - American Crocodile American Blue Rabbit MC Question: Given the table above, correctly give the order of each species on a cladogram that is read from left to right. American Crocodile, American Blue Rabbit, European Tree Frog American Blue Rabbit, European Tree Frog, American Crocodile European Tree Frog, American Blue Rabbit, American Crocodile American Crocodile, European Tree Frog, American Blue Rabbit Table 2: Species Lungs Claws or Nails Feathers Fur; mammary glands A + - B C D E FRQ: Given the table, construct a cladogram including all of these species. Then, explain what it means in terms of the evolutionary process and speciation. Provide examples of possible species for species A-E. In terms of the evolutionary process, both the cladogram and the given table recognize that a salamander came first. It then closely shares a common ancestor with the lizard who through either allopatric (geographic/physical) or sympatric (chromosomal) speciation gained either claws or nails. The lizard shares a close common ancestor to the pigeon who, throughout time and speciation, picked up alleles for feathers and fur/mammary glands. Following the pigeon in the evolutionary process is the mouse who shares all the common traits as the pigeon minus the feathers. The mouse then shares a close common ancestor with a chimp. Reading the cladogram from left to right you can determine the evolutionary process of the species. Examples: A: Salamander; B: Pigeon; C: Mouse; D: Lizard; E: Chimp. E D C B A Feathers Fur; mammary glands Claws or Nails lungs

26 Which of the following is false about sensory neurons?
LO 3.50: The student is able to create a visual representation to describe how the vertebrate brain integrates information to produce a response. SP 1.1: The student can create representations and models of natural or man-made phenomena and systems in the domain. Explanation: Three types of neurons use electrical impulses to detect stimuli and transfer them into responses. Sensory neurons in the peripheral nervous system detect external stimuli and transmit that impulse to the spinal cord and brain in the central nervous system. The information is integrated with the help of interneurons. Lastly, motor neurons send a response to the rest of the body. Motor neurons trigger muscle or gland activity. Gland activity is started by the hypothalamus that is in the brain that detects changes in hormonal activity and sends signals to the pituitary gland, which controls all of the other glands to produce a hormonal response. MCQ: Which of the following is false about sensory neurons? A. They could detect the warmth of a flame. B. They are protected by bone. C. The sensory receptor can be the afferent neuron in the pathway. D. The sensory neuron must reach a threshold in order to send the stimulus. FRQ: Propose you are driving and you realize a police officer is behind you and signaling you to pull over. Describe your body's first reaction to the fear, be sure to include the types of neurons used, two physiological reactions as well as two glands that would respond to the stimulus. Draw a simple pathway to show the pathway from stimulus to response.

27 Answer Key MCQ: The correct answer is B. The sensory neurons are part of the peripheral nervous system and only the central nervous system is protected by bone. The CNS includes the brain and spine. FRQ: Sensory neurons from the eyes would send an electrical impulse to the interneurons. The interneurons located in the spine and brain would recognize the stimulus of fear. Motor neurons that trigger muscles and gland activity would dilate the pupils and increase your heart rate. This was done by the pituitary gland which acts as the "master gland" and instructs the other glands in the body to secrete hormones. The pituitary gland would make the adrenal glands secrete epinephrine, an excitatory neurotransmitter, that is given when a person feels intense fear, like being pulled over.

28 Free-response question
(LO 4.11)The student is able to justify the selection of the kind of data needed to answer scientific questions about the interaction of populations within communities. (SP 1.4)The student can re-express key elements of natural phenomena across multiple representations in the domain. (SP 4.1) The student can justify the selection of the kind of data needed to answer a particular scientific question. Explanation A community is classified as many different species close enough to possibly interact. There are many different types of interactions which include competition, perdition, herbivory, symbiosis, and disease. Interspecific competition is when species compete for limited resources (e.g. grasshoppers and bison compete for grass). Predation is a interaction between species in which one species kills and the other is being eaten by the other (e.g. lion and antelope). Herbivory is the interaction between an herbivore and a plant (e.g. a cow and grass). Forms of symbiosis include mutualism, commensalism, and parasitism. Mutualism is an interaction where both of the species benefit. Commensalism is where one species benefits from the interaction but the other isn’t harmed. Parasitism is the interaction in which a parasite uses a host to get its nutrients while harming the host (e.g. ticks). Disease is when pathogens which are disease-causing agents attack on host organisms (e.g. influenza). Multiple choice All the following are examples of how predators can adapt to better Atch their pray except which one? wolfs adapt sharper claws to kill their prey quicker Hawlmoth larva adapts mimicry and mimics a harmful snake to catch its prey cheetahs adapts speed and agile to better catch up with its prey Florida alligator camouflages into its surroundings to surprise its prey Free-response question Describe how interspecific competition has affected the species according to the figure provided, and list other possible results that could have occurred because of interspecific competition.

29 Answer key All the following are examples of how predators can adapt to better catch their pray except which one? wolfs adapt sharper claws to kill their prey quicker Hawlmoth larva adapts mimicry and mimics a harmful snake to catch its prey cheetahs adapts speed and agile to better catch up with its prey Florida alligator camouflages into its surroundings to surprise its prey (B) is the correct answer because (A), (C), and (D) are all adaptations a predator can adapt to facilitate the capture of its prey with claws, speed, and camouflage. In answer choice (B) is talking about a non-harmful species mimicking a harmful creature to capture its food, but the mimicry will only benefit the species by avoiding predation. Describe how interspecific competition has affected the species according to the figure provided, and list other possible results that could have occurred because of interspecific competition. In the first two graphs, both of the species are increasing and growing when they are cultivated separately, and dying naturally or other unknown reasons. In the third graph, one of the species are prospering and growing at their normal rate while the other species are at first increasing, but then diminishes in number and finally all of the species die out. Which demonstrates because of limited resources in short quantity both species competed, but one species has an advantage and survived to reproduce. Other possible results could have been that one of the species can find another niche in order to survive, or the species could have better adapted to be able to compete for the resource.

30 Model excitatory pathway Model Inhibitory pathway
LO 3.33: The student is able to use representation(s) and appropriate models to describe features of a cell signaling pathway. SP 1.4: The student can use representations and models to analyze situations or solve problems quantitatively or qualitatively. Explanation: Cell signaling can occur locally from cell to cell (paracrine signaling), using chemical signals transmitted through structures like Plasmodesmata or vesicles, or by long distance pathways (endocrine signaling) via chemical signals transported through the bloodstream, to produce or modify a cellular response or process through signal transduction. Cells can secrete Ligands, or chemical signal molecules (including hormones and other chemical messengers) that activate the beginning of cell signaling pathways, which come in two general forms; the first one being molecules incapable of crossing the plasma membrane that bind to integral proteins on the cell membrane; and then there are also steroid ligands that go through the cellular membrane and directly initiate a cellular response on the inside of the cell (DNA expression or enzyme activity). The ligands that bind to receptor sites with complementary shapes on the extracellular part of an integral protein will then change the shape and function of this receptor protein. The three main receptor proteins are: G-protein-linked receptors, that phosphorylate G-proteins so they can activate enzymes that cause cellular responses, Tyrosine Kinases, that cause a chain (cascade) in relay protein phosphorylation, which are used as second messengers, to send a signal to its correct destination, and then there are also Ion channels that open up when activated by a ligand and let ions into the cell resulting in a cellular response. The destination of the signal changes depending on the cell and the proteins (receptors and second messengers) activated so it may be directed to a specific structure within the cell, or to the DNA. Second messengers are the molecules or ions inside of a cell that relay signals in a signal transduction pathway to the final destination. There are several important second messengers used in several different transduction pathways such as G-proteins and relay proteins, that send signals through phosphorylation, cyclic AMP (Adenosine Mono-Phosphate) that acts as an allosteric activator for enzymes that produce ATP (for the phosphates used for phosphorylation), and Ca2+ ions, which are involved in muscle contraction and also IP3, which is second messenger derived from the plasma membrane that is used to release Ca2+ ions into the cytosol. When a cell receives a signal or multiple signals, it can be amplified to result in hundreds of thousands of simultaneously occurring transduction pathways that will make the cellular response (DNA response, enzyme activity, production of something, inhibition of a cellular action or process etc.) more efficient. M.C. Question: Muscle contraction occurs when motor neurons release a neurotransmitter (ligand) Across a synapse to a muscle fiber. The muscle fiber cells then transduce the signal and it results in their contraction. While exercising, John’s skeletal muscle cells are being signaled to contract repeatedly while he lifts weights. What are the two 2nd messengers that are most important to the Contraction of muscles? Choose the best answer. (Use figure 1.1 and your knowledge to answer this question) cAMP and Ca2+ IP3 and Rho Kinase cGMP and ATP Ca2+ and IP3 Free Response Question Figure 1.1: The signaling pathway for a muscle contraction (+), and the relaxation (-) Cellular responses to signal transduction pathways can be very complicated and a cellular Response is often controlled by more than 1 pathway, or combinations of pathways. One combination could be excitatory and inhibitory pathways. These two pathways work together to start and stop cellular actions and processes. Suppose that there is a signal transduction pathway where a ligand binds to an tyrosine kinase receptor to begin the production of a secreted protein, and there is a steroid ligand that inhibits that production by binding to the enzyme that transcribes for that protein. Suppose you know an individual who overproduces this protein and cannot stop its production without medicinal help. Describe the signal transduction pathways for the excitatory pathway and the inhibitory pathway of this process and then explain what could be the reason for the overproduction of this protein in this individual as it relates to signal transduction pathways. (use to figures next to question for help) Model excitatory pathway Model Inhibitory pathway

31 Answer to multiple Choice
M.C. Question: Muscle contraction occurs when motor neurons release a neurotransmitter (ligand) Across a synapse to a muscle fiber. The muscle fiber cells then transduce the signal and it results in their contraction. While exercising, John’s skeletal muscle cells are being signaled to contract repeatedly while he lifts weights. What are the two 2nd messengers that are most important to the Contraction of muscles? Choose best answer. (Use figure 1.1 and your knowledge to answer this question) cAMP and Ca2+ IP3 and Rho Kinase cGMP and ATP Ca2+ and IP3 Answer to Free Response Two inactive tyrosine Kinase proteins will receive a ligand molecules that will bind to their receptor sites. The two proteins then join together to make an active tyrosine kinase protein that will phosphorylate relay proteins to use as second messengers and then each relay protein will phosphorylate the next relay protein in the chain until it goes all the way down the phosphorylation cascade. When it reaches the bottom of the cascade, the final protein, an enzyme will go into the nucleus and begin to transcribe mRNA for the secretory protein that is to be made. When it is time to stop transcribing for this protein, a steroid ligand will pass through the plasma membrane and into the nucleus where it will bind to a complementary site on the enzyme, changing its shape and rendering it useless. This individual has a problem in his inhibitory signal transduction pathway. A possible problem in this pathway could be that there is a mutation in the enzyme that transcribes for the protein and it does not have the correct complementary shaped site for the hormone to bind to so it can have its inhibitory effects, therefore not letting the production of this secretory protein come to a stop and overproducing it.

32 LO 2.16 The student is able to connect how organisms use negative feedback to maintain their internal environments. SP 7.2 The student can connect concepts in and across domain(s) to generalize or extrapolate in and/or across enduring understandings and/or big ideas. Explanation: Organisms maintain homeostasis using negative feedback loops. In a negative feedback loop, the response is in the opposite direction of the stimulus. A target amount, temperature, etc. is desired. For example, humans use a negative feedback loop to maintain an optimal body temperature of 98.6 F, which is called thermoregulation. Receptors in the hypothalamus and on the skin monitor the temperature of the body. When it is too hot, actions such as sweating are initiated to cool the body. When it is too cold, actions such as shivering are initiated to warm the body. This is indicated in Figure 2.16, seen below. Another example is operon regulation of genes in bacteria. Since bacteria live in harsh conditions in which their environments change quickly, their genes need to be turned off or on quickly. The trp operon is a series of genes in bacteria that are activated when bacteria need tryptophan, an essential amino acid. When trp levels are high in the bacteria, a repressor protein is activated, which binds to the trp operon. This prevents transcription of the DNA that codes for the enzymes that make tryptophan. When trp levels lower, transcription begins again. This is a negative feedback loop because high levels of tryptophan prevent more tryptophan from being made. Multiple Choice: Which of the following is a negative feedback loop? A) Childbirth B) Regulation of water levels in plants C) The ripening of fruit D) Lactation in mammals FRQ: Organisms use negative feedback loops to maintain homeostasis. Define homeostasis. Then, discuss how humans regulate internal blood glucose levels through a negative feedback loop. Be specific in your response. Figure 2.16

33 Answer Key Correct Answer: B
A. Since the baby’s head pushing causes the cervix to stretch and more of the hormone oxytocin to be released and the cycle continued, childbirth would be a positive feedback loop, thus A is incorrect. B. The secretion of hormones to regulate metabolic rate requires negative feedback between the anterior pituitary and the thyroid, thus B is correct. C. Since the presence of ethylene in one fruit causes those fruits around it to ripen and more ethylene to be produced, which represents a positive feedback loop, C is incorrect. D. Since suckling by the baby causes the release of oxytocin, which leads to more milk secretion, which is a positive feedback loop, D is incorrect. FRQ Homeostasis is a tendency to stability in an organism. In humans, the regulation of blood glucose levels represent a negative feedback loop. The pancreas uses a glucose receptor cell to monitor blood glucose levels. The α-cells in the pancreas produce insulin while the β-cells produce glucagon. When glucose levels are too high, insulin stimulates glucose to be converted to glycogen, thus lowering blood glucose levels. When glucose levels are too low, glucagon stimulates the breakdown of glycogen to glucose to raise blood glucose levels.

34 Learning Log/FRQ-style Question:
LO 3.18: The student is able to describe the connection between the regulation of gene expression and observed differences between different kinds of organisms SP 7.1: The student can connect phenomena and models across spatial and temporal scales Explanation: Although every cell in an organism has the same DNA sequences within, each cell expresses certain genes a little differently. Gene expression is mainly regulated through transcription, RNA processing and nuclear export, translation, and post translation that allow polypeptide proteins to be developed. It can also be regulated through the amplification, deletion, or rearrangement of certain segments, although this is rare. Transcription of specific genes occurs as polymerase binds to DNA with the help of transcription factors that then aid in the production of an RNA strand that can be used with it. During RNA processing, splicing, or exons being joined as introns are removed, which can also alter the way certain genes are expressed within a cell. Following this, mRNA is transported into the cytoplasm, where it is translated and forms a resulting polypeptide product, or protein that can then be used throughout the cell. In terms of physical expression, the genes can be shown as either genotypes (a combination of alleles situated on specific chromosomes that make up genetic traits) and phenotypes (physical or observable characteristics of an organism). Some examples of the different expressed genes in various organisms include fur in certain animals, flower production in certain plants, and the ability of certain bacteria to regulate protein production within larger organisms. Gene expression varies in these different types of organisms due to their varying cellular structure and DNA composition. M.C. Question: Which of the following statements regarding the process of the regulation of gene expression is true? A) Splicing, which is defined as exons being removed and introns being joined, happens during RNA processing B) During RNA processing, mRNA is transported to the mitochondria C) Gene amplification and deletion that occurs prior to translation is very common D) Transcription of specific genes occurs as polymerase binds to DNA E) Gene expression is primarily regulated through transformation Learning Log/FRQ-style Question: Describe how the regulation of gene expression may differ in eukaryotes and prokaryotes, and explain the process that causes gene expression to occur.

35 ANSWER KEY: LO 3.18 M.C. Question: Which of the following statements regarding the process of the regulation of gene expression is true? A) Splicing, which is defined as exons being removed and introns being joined, happens during RNA processing B) During RNA processing, mRNA is transported to the mitochondria C) Gene amplification and deletion that occurs prior to translation is very common D) Transcription of specific genes occurs as polymerase binds to DNA E) Gene expression is primarily regulated through transformation Learning Log/FRQ-style Question: Describe how the regulation of gene expression may differ in eukaryotes and prokaryotes, and explain the process that causes gene expression to occur. Since eukaryotes are made up of multiple cells and contain complex, membrane bound organelles, and a nucleus, their ability to express genes is different from that of prokaryotes, which are only made of one cell, lack a nucleus, and do not have membrane bound organelles. In eukaryotes, all of the cells in the organism contain DNA and RNA housed in the nucleus, meaning that they all have genetic information within them. It is then up to the cell, through the process of gene regulation, to determine which genes will be used/turned on in specific cells. The turning on of cells occurs through the genetic information within the nucleus going through transcription RNA splicing, export to the cytosol, and translation into a protein. Prokaryotes also contain large amounts of DNA and RNA, although it is found in a very long strand, not in a nucleus. Differently from eukaryotic gene expression, transcription and translation can occur at the same time, while in eukaryotes transcription must precede translation. Prokaryotes also have the ability to regulate how much transcription occurs, thereby determining how much RNA is able to be produced and translated for protein production.

36 LO1.1: The student is able to convert a data set from a table of numbers that reflect a change in the genetic makeup of a population over time and to apply mathematical methods and conceptual understandings to investigate the cause(s) and effect(s) of this change. SP 1.5: The student can re-express key elements of natural phenomena across multiple representations in the domain. SP 2.2: The student can apply mathematical routines to quantities that describe natural phenomena. Explanation: Natural selection is the driving force behind evolution, and genetic variation is the driving force behind natural selection. A population evolves, not an individual; therefore, we measure genetic variation based on changes in the population’s gene pool (all of the genes in a population at any time, including alleles). Natural selection acts on genetic variation such that animals more “fit” for their environment survive and reproduce to make offspring that are better suited to their environment (Reproduction of the Fittest). The Hardy-Weinberg Theorem is what describes a non-evolving population in which the alleles and genotypes remain constant unless acted upon b agents other than sexual recombination. It is these agents which we examine because actual populations are rarely if ever at Hardy-Weinberg equilibrium (this constancy of ratios). To maintain this equilibrium, a population must fulfill the five key requirements: Large population size; Isolation; No net mutation; Random mating (not based on any preference for certain traits); No natural selection (prevalence of certain traits due to selective pressures). If any of these requirements are not met, then microevolution is occurring within the population. Hardy and Weinberg used the letters p and q to denote the proportions of dominant and recessive alleles in a population. 𝑝+𝑞=1 describes the phenotypic proportions, or frequencies, in the population: dominant and recessive, respectively. 𝑝^2+2𝑝𝑞+𝑞^2=1 describes the genotypic proportions, or frequencies, in the population: homozygous dominant, heterozygous, and homozygous recessive, respectively. The equations must add to 1 because the frequencies represent 100% of the population. Using these equations, we can determine how a population has changed over time. If the population has not evolved (that is, it is in Hardy-Weinberg equilibrium), then the frequencies at the beginning of a study on the population should match those taken at the end of the study. The graph at right shows the relationship between the allelic frequencies in the Hardy–Weinberg theory. The horizontal axis shows the two allele frequencies p and q and the vertical axis shows the expected genotype frequencies. Each line shows one of the three possible genotypes within the population. MC QUESTION: There are 3754 orchids in a large greenhouse that are allowed to pollinate freely. Using the table given, what is the genotypic frequency for the heterozygous plants? .6505 .4232 .4547 .3495 FRQ QUESTION: You are a scientist studying a population of 1400 freshwater fish in a lake given to frequent agitation by environmental factors, such as rainfall and earthquakes. The data given at right are averages accumulated every year from 1950 to 2010. a) Suggest two causes for the pattern you see and explain your reasoning in detail using words and numerical evidence. Conclude whether the fish are evolving or not. b) In 1993, an invasive, predatory species of bottom-dwelling fish is introduced accidentally to the lake. The number of brown fish in 2000 is 670 and in 2010 is 643 while the numbers of blue fish increase slightly. Describe what may have occurred using words and numerical evidence. Predict the trend in future years. Color Count Red 2442 Orange 1312 Years # Brown # Blue Years c. # Brown c. # Blue c. 1950 668 732 1990 724 676 1960 674 726 2000 760 640 1970 718 682 2010 781 619 1980 752 648 Years (part b) # Brown (part b) #Blue (part b) 2000 670 650 2010 643 679

37 http://en.wikipedia.org/wiki/Hardy%E2%80%93Weinberg_principle – Credit for the graph
ANSWERS MC QUESTION: There are 3754 orchids in a large greenhouse that are allowed to pollinate freely. Using the table given, what is the genotypic frequency for the heterozygous plants? .6505 – this is wrong because it is the frequency for the dominant phenotype (p=2442/3754=.6505). .4232 – this is wrong because it is the frequency for the homozygous dominant genotype (p^2=.6505*.6505=.4232). .4547 – This is the correct answer [2(.6505)(.3495)=.4547]. .3495 – this is wrong because it is the frequency for the recessive phenotype (q=1312/3754=.3495). FRQ Rubric: a) Credit for stating the pattern observed, naming two Hardy-Weinberg requirements for equilibrium and stating how they are being violated, and for positing how those violations are possible causes for the pattern observed. Additional credit for stating that individual fish do not evolve. Must give mathematical evidence to support claims (the Hardy-Weinberg equations). b) Credit for stating the trend change, predicting future change, and linking the trend to one or more Hardy-Weinberg requirements. Sample Answer: a) Overall, the pattern in the population is that the brown phenotype is increasing in frequency and the blue frequency is decreasing in frequency. One possible explanation for this is that the brown phenotype is being selected for by mates as a more desirable trait, which violates the Hardy-Weinberg requirement of random mating. Another possible explanation is that the silt on the bottom of the lake is more like clay and therefore is more brown. The fish could be hiding from predators at the bottom of the lake, which is a selective pressure and means that the fish population is microevolving in response to its predators, which violates the Hardy-Weinberg requirement stating that no Natural selection can take place. It is clear that the fish population (not the individual fishes) is evolving to favor the brown phenotype over the blue phenotype. The mean number of brown fish is about 725, and the mean number of blue fish is about 675. Using the Hardy-Weinberg equation for phenotypes, the frequency of brown fish (.5179) is greater than the frequency for blue fish (.4821). b) Natural selection is taking place against the brown fish phenotype presumably because the predator finds it easier to catch the brown fish, which may get their coloration from ancestors that historically hid in the muddy waters closer to the bottom of the lake. The blue fish will be better able to survive and reproduce because their coloration makes them less susceptible to predation by the invasive, bottom-dwelling species. Unless the invasive species is removed, the brown phenotype will continue to decrease in favor of the blue coloration which is better suited for the rest of the lake and/or a behavioral change (i.e. not hiding at the bottom of the lake) and which would result in the survival and reproduction of offspring more fit for the new predator in their environment.

38 SP 6.1 The student can justify claims with evidence
LO 2.39 The student is able to justify scientific claims, using evidence, to describe how timing and coordination of behavioral events in organisms are regulated by several mechanisms. SP 6.1 The student can justify claims with evidence Explanation: Used in hibernation, estivation, migration and courtship, behaviors in animals are triggered by environmental cues, for example, in photoperiodism in plants, changes in the length of night regulate flowering and preparation for winter. These behaviors are vital to reproduction, natural selection and survival, for example, in phototropism in plants, changes in the light source lead to differential growth, resulting in maximum exposure of leaves to light for photosynthesis. This demonstrates the plant acting in a behavior that is vital for survival. In this Powerpoint we will focus on courtship. These behaviors are genetically and environmentally caused. Genetic components are the sounds animals make to court their mates, scents and smells they give off and phenotypic traits, for example the size of antlers can contribute to if you court a mate, the bigger the antlers the more in demand you are. Environmentally caused behaviors are times when animals reproduce, and when they are in heat. On the right we have a figure which shows fruit flies courtship behaviors. MC Question: Which of the following regarding animal courtship behaviors is untrue? A) The goal of courting a mate is to reproduce. B) Certain animals attract their mate with dancing. C) Animals courtship behaviors are not genetically caused. D) Phenotypic traits do not set you apart. Learning Log/FRQ-style question: Why would it be beneficial for an animal to avoid reproducing in the winter? State a specific environmental cause as to why they avoid reproducing during that time in your answer. How has this increased the offsprings rate of survival and lead to surviving natural selection? Date accessed 4/27

39 Answer Key- LO 2.39 Which of the following regarding animal courtship behaviors is untrue? A) The goal of courting a mate is to reproduce. B) Certain animals attract their mate with dancing. C) Animals courtship behaviors are not genetically caused. D) Phenotypic traits do not set you apart. Why would it be beneficial for an animal to avoid reproducing in the winter? State a specific environmental cause as to why they avoid reproduce during that time in your answer. How has this increased the offsprings rate of survival and lead to surviving natural selection? It is beneficial for an animal to avoid reproducing in the winter because the temperature is cold and it is hard enough to find food at is for just one animal so finding food for another animal is even more difficult. The environmental cause would be because of the colder temperature. This increases the rate of survival because the offspring will have a better chance of surviving since they will not be born in the winter when food is scarce. This has lead to surviving natural selection because the offspring and the animal survive since the environment will be able to support them.

40 A.) The neuron would not be able to travel as quickly down the axon.
LO 3.45: The student is able to describe how nervous systems transmit information. SP 1.2: The student can describe representations and models of natural or man-made phenomena and systems in the domain. Explanation: The basic structure for the nervous system is the neuron, and the anatomy of the neuron is key in the transmission of information throughout the nervous systems. The neuron consists of the cell body, the axon, and the dendrites. Along with these key components, the neuron also contains the axon hillock and the synapse. The axon hillock generates the neural impulses and is the start of the axon. The axon, which is responsible for taking the signal away from the cell body, is insulated by the myelin sheath, and is made of supporting cells - Schwann cells secrete myelin in the peripheral nervous system and oligodendrocytes secrete myelin in the central nervous system. The myelin sheath, however, is not continuous, but has space between Schwann cells along the axon called the nodes of ranvier. The nodes of ranvier aid in speeding up neuron firing along the synapse, which is the junction between the neuron and the next cell - whether it be a neuron or a muscle cell. There are two different types of neurons, afferent (sensory) neurons and efferent (motor) neurons. Neurons are negatively charged, and the charge is -70mV. There is more potassium inside of the neuron, and sodium outside, and there are many sodium and potassium channels along the neuron. When potassium channels open, more potassium is let inside of the neuron, and the neuron becomes more negatively charged (hyperpolarized). When sodium channels open, sodium enters, and the neuron becomes less negative (depolarized). If enough sodium enters the neuron, making the neuron depolarized to the threshold of -55 mV, then an action potential fires. Action potential reproduces the impulses along the neurons, and are needed for transmission. Actual transmission, however, occurs across the synapses. This transmission firing usually takes place with neurotransmitters (chemical messengers). Firstly, dendrites of the presynaptic neuron receive a stimulus from either another neuron or from a sensory cell that detects a stimulus from the environment. This stimulus travels from the dendrites to the axon hillock, which fires the impulse in one direction down the axons. The myelin sheath and nodes of ranvier speed along the firing process. As it speeds down the axon, it connects with the dendrites of the postsynaptic neuron, and the cycle repeats itself. Eventually, the impulse reaches the central nervous system, containing the spinal cord and brain, and the information is received and analyzed. Then, a response is sent back down the dendrites, axon hillock, and axons, and a response is generated. AP Style Question: If a neural impulse traveled down the presynaptic neuron and reached the postsynaptic neuron, but the postsynaptic neuron did not contain an axon hillock, which of the following would occur? A.) The neuron would not be able to travel as quickly down the axon. B.) The neural impulse would not fire. C.) The neural impulse would not be detected. D.) The neural impulse would travel back down the presynaptic neuron. FRQ Style Question: The picture to the right depicts a functioning motor neuron, capable of receiving and transmitting neural impulses. A.) Identify and describe the key components of the neuron, paying special attention to the parts of the diagram that are emphasized with arrows. B.) Amyotrophic lateral sclerosis, or Lou Gehrig’s disease, is a neurodegenerative disease that affects nerve cells in the brain and the spinal cord and degenerates motor neurons in the central nervous system. Describe the effects amyotrophic lateral sclerosis would have on the body and the nervous system.

41 Answer Key AP Style Question: If a neural impulse traveled down the presynaptic neuron and reached the postsynaptic neuron, but the postsynaptic neuron did not contain an axon hillock, which of the following would occur? A.) The neuron would not be able to travel as quickly down the axon. B.) The neural impulse would not fire. C.) The neural impulse would not be detected. D.) The neural impulse would travel back down the presynaptic neuron. FRQ Style Question: The picture to the right depicts a functioning motor neuron, capable of receiving and transmitting neural impulses. A.) Identify and describe the key components of the neuron, paying special attention to the parts of the diagram that are emphasized with arrows. B.) Amyotrophic lateral sclerosis, or Lou Gehrig’s disease, is a neurodegenerative disease that affects nerve cells in the brain and the spinal cord and degenerates motor neurons in the central nervous system. Describe the effects amyotrophic lateral sclerosis would have on the body and the nervous system. Sample Answer: A.) A neuron consists of the dendrites, which detect stimulus from either other neurons or from sensory cells, which are cells that detect stimulus from the environment around them. A neuron also contains a cell body, which houses the nucleus, or the control center from the neuron itself. An important component of the cell body is the axon hillock, which generates an impulse and begins the axon. The axon takes the signal away from the cell body. The axon is covered in myelin, which is a sheath that insults the axons to make the signal travel faster down the neuron, and is composed of supporting cells, such as Schwann cells in the peripheral nervous system and oligodendrocytes in the central nervous system. Finally, the neuron has the synapse, which is responsible for transmission between neurons, as it is the junction between the neuron and the next cell. B.) Amyotrophic lateral sclerosis would cause voluntary muscle movement to eventually cease. The motor neurons would no longer be able to detect, receive, analyze, or transmit information. So, the brain could no longer initiate muscle movement nor control it. The patient would become paralyzed. The muscles would weaken significantly and become degraded, so movement, speech, swallowing, and breathing would be hindered. Lou Gehrig’s Disease would eventually lead to death of the victim.

42 LO 2.6: The student is able to use calculated surface area-to-volume ratios to predict which cells might eliminate wastes or procure nutrients faster by diffusion. Nerve cells are long and thin thus giving them a high SA-to-V ratio. SP 2.2: The student can apply mathematical routines to quantities that describe natural phenomena. Explanation: As the size, or volume, or a cell increases, its surface area will increase as well. What is most ideal for a cell, in terms of its metabolic rates, is when the surface area is higher than that of the volume. This allows for more total space to allow substances to pass into and out of the cell via the plasma membrane at a faster and more efficient rate in order to meet the needs of the cell. The volume of a cell grows at a quicker rate than that of the SA, thus, the smaller the object, the greater its surface area-to-volume ratio which is more efficient for a cell. The volume being at a lesser size than the surface area is also positive because the larger the cells is, the more nutrients will need to be procured or wastes needed to be eliminated. The higher the surface area-to-volume ratio, the better it is for the cell. Some cells have microvilli extending off of the cell membrane (see FRQ). Ratio of Cell A Ratio of Cell B A. 1.2 B. 3 4 C. 1 D. 2.2 SA sphere: 4π 𝒓 𝟐 V Sphere: 𝟒 𝟑 π 𝒓 𝟑 SA cube: 6 𝒔 𝟐 V cube: 𝒔 𝟑 MC Question: By using the formulas to the right, use your knowledge of surface area-to-volume ratios of find the ratio of the following two types of cells: Cell A: spherical cell: radius of 3 Cell B: cuboidal cell: side length of 5 FRQ: By using your answer from the multiple choice question, predict which cell, A or B, would be overall more efficient? Identify one benefit and one challenge of each cell in terms of the SA-to-V ratio. Also discuss whether it would be an advantage or disadvantage to having microvilli (as seen in the photo above) extending off of the cell membrane. Explain.

43 Cell A: spherical cell: radius of 3
MC Question: By being given the formulas, use your knowledge of surface areas-to-volumes ratio of find the ratio of the following two types of cells. Cell A: spherical cell: radius of 3 Cell B: cuboidal cell: side length of 5 Ratio of Cell A Ratio of Cell B A. 1.2 B. 3 4 Answer: C. 1 D. 2.2 SA sphere: 4π 𝒓 𝟐 V Sphere: 𝟒 𝟑 π 𝒓 𝟑 SA cube: 6 𝒔 𝟐 V cube: 𝒔 𝟑 In this flat skin cell, the entire SA is in contact with the environment, so it has a high SA-to-V ratio. FRQ: By using your answer from the multiple choice question, predict which cell, A or B, would be overall more efficient? Identify a one benefit and one challenge of each cell in terms of the SA-to-V ratio. Also discuss whether it would be an advantage or disadvantage to having microvilli (as seen in the photo) extending off of the cell membrane. Justify and explain. FRQ answer: Cell B, the cuboidal cell, would be more efficient because it has a higher surface area to volume ratio than Cell A. Cell B would thus have the benefit of having more total area to let substances pass into and out of the cell which makes the process of diffusion more efficient A possible challenge could be that since there is more total SA (relative to the volume), water and other dissolved substances are lost more quickly which can cause a problem like dehydration for the cell. Also temperature control and regulation could be a issue because the higher SA causes more means for heat to escape. Likewise one the few advantages to cell A is that, because it has a lower SA-to-V ratio, there is less total area (relative to the volume) for water and things that are actually needed in the cell to leave. The challenges of cell A, however, is that its rate of diffusion would be less effective because there is less total area for nutrients/ wastes to exit but the higher volume means that the cell as whole requires more nutrients/wastes. It would definitely be an advantage to have the microvilli extending off of the plasma membrane because they would greatly INCREASE the total surface area of the cell where diffusion can take place, while also causing very LITTLE increase in the volume because of their skinny size. Microvilli would be useful in the absorption and secretion of materials because it makes the process of diffusion quicker and more effective by increasing the SA-to-V ratio.

44 LO 4.27: The student is able to make scientific claims and predictions about how species diversity within an ecosystem influences ecosystem stability. SP 6.4: The student can make claims and predictions about natural phenomena based on scientific theories and models. Explanation: Ecosystems with less biodiversity and less complexity (fewer operative parts) are less resilient to changes in the environment, making them more fragile than those more diverse and complex. The diversity of an ecosystem relies on trophic levels- producers and various (primary, secondary, and tertiary) consumers, abiotic factors, such as wind, sunlight, climate, biotic factors, such as the eating habits (carnivore v. herbivore) of other species, and keystone species. The ecosystem relies on the continued presence of the keystone species, who have an enormous effect on the interactions and relationship within the ecosystem. Learning Log/FRQ-style Question: Consider the case of the ecosystem described in Figure (pg. 1169). Sea urchins feed on kelp, and sea otters feed on sea urchins. Where sea otters are abundant, few sea urchins are around, and the kelp forests thrive. In areas where there are few otters, sea urchins dominate and the kelp forests are very underdeveloped. Over the last 20 years, killer whales have been preying on sea otters because their natural prey has become less abundant, causing their population to decrease significantly, leaving the sea urchin population unchecked and decimated kelp forests. In this scenario, identify the primary, secondary, and tertiary consumers, and a possible keystone species in this ecosystem. Predict the future of this ecosystem without the presence of the sea otter, and suggest one possible remedy to treat the results described in your prediction. Multiple Choice: A new wind farm is being installed in a valley in the Blue Ridge Parkway. Upon arriving to the site, construction workers notice an area near the valley sparsely vegetated, with most plants leaning in a specific direction, and are suspicious of whatever environmental factor is causing this unusual growth pattern. Of the following choices, the best possible explanation of this pattern is The non-uniform distribution of sunlight on the earth’s surface An extreme temperature difference between the shady side of the valley and the side near continuously lit by the sun A lack of natural herbivorous predators in the ecosystem A strong, steady force of wind in the valley continuously blowing in a particular direction

45 Answer Key- LO 4.27 Q:Consider the case of the ecosystem described in Figure (pg. 1169). Sea urchins feed on kelp, and sea otters feed on sea urchins. Where sea otters are abundant, few sea urchins are around, and the kelp forests thrive. In areas where there are few otters, sea urchins dominate and the kelp forests are very underdeveloped. Over the last 20 years, killer whales have been preying on sea otters because their natural prey has become less abundant, causing their population to decrease significantly, leaving the sea urchin population unchecked and decimated kelp forests. In this scenario, identify the primary, secondary, and tertiary consumers, and a possible keystone species in this ecosystem. Predict the future of this ecosystem without the presence of the sea otter, and suggest one possible remedy to treat the results described in your prediction. Answer: Primary- sea urchin, secondary- sea otter, tertiary- killer whale. The otter is the keystone species, because so many other species depend upon the survival of the sea otter. Without the predation by the otter, the kelp forests will be destroyed by the sea urchins. Because they are the producers mentioned in this trophic chain, no other species will be able to survive if the kelp forests can’t survive, who directly depend on the sea otter. The sea otter has an enormous effect on its ecosystem in comparison to its relatively small presence in the ecosystem. Without the sea otter, the ecosystem will eventually collapse as the killer whale will have nothing to eat, and the sea urchin population will continue to grow without a direct predator, destroying its food source in turn. A possible remedy would be to introduce a new species of sea kelp that sea urchins cannot eat, increasing the producer level, and therefore increasing the stability of the ecosystem, or to reintroduce the killer whale’s natural prey to decrease its dependence on the sea otter population for survival. MCQ:A new wind farm is being installed in a valley in the Blue Ridge Parkway. Upon arriving to the site, construction workers notice an area near the valley sparsely vegetated, with most plants leaning in a specific direction, and are suspicious of whatever environmental factor is causing this unusual growth pattern. Of the following choices, the best possible explanation of this pattern is a) The non-uniform distribution of sunlight on the earth’s surface b) An extreme temperature difference between the shady side of the valley and the side near continuously lit by the sun c) A lack of natural herbivorous predators in the ecosystem d) A strong, steady force of wind in the valley continuously blowing in a particular direction

46 3.5- Justify the claim that humans can manipulate heritable information by identifying at least two commonly used technologies. SP and some Science Practice 6.4 The student can make claims and predictions about natural phenomena based on scientific theories and models. Explanation heritable information is in humans stored in DNA and manipulated in a plethora of ways i.e. using restriction enzymes that recognize specific codons and will splice the DNA at that site allowing for the transfer of that gene onto a plasmid that can then be inserted into a host cell, where the gene will again be spliced into the hosts cell genetic material. Or the gene along with its plasmid will use the host cells resources to replicate within it and during mitosis passing through the generations (see figure 1). if the gene is spliced into the genome of the host cell then gene expression will take place i.e. insulin secretion. gene splicing (figure 2). MC question Based on what you know about viruses, DNA replication and splicing, predict the capabilities of a virus if scientists replaced virus DNA with DNA fragments of interest, then injected them into the bloodstream. A) White blood cells would immediately destroy all the viruses before any action by the viruses could occur B) The virus would self repair its DNA with its own restriction enzymes and infect cells C) The Virus would inject the genetic material into host cells allowing plasmid reproduction or gene expression to occur D) The virus would not be able to function FRQ A) Describe the process of genetic splicing of a specific gene into a host organism at the cellular level. B) And give an example of the uses of splicing genetic information into host cells. Figure 1 Figure 2

47 Answer key MC question Based on what you know about viruses, DNA replication and splicing, predict the capabilities of a virus if scientists replaced virus DNA with DNA fragments of interest, then injected them into the bloodstream. A) White blood cells would immediately destroy all the viruses before any action by the viruses could occur B) The virus would self repair its DNA with its own restriction enzymes and infect cells C) The Virus would inject the genetic material into host cells allowing plasmid reproduction or gene expression to occur D) The virus would not be able to function FRQ A) Describe the process of genetic splicing of a specific gene into a host organism at the cellular level. B) And give an example of the uses of splicing genetic information into host cells. A) First a restriction enzyme specific to the  gene being spliced must cut it from the DNA strand, once it has  formed a plasmid the genetic material can either be placed in a vector (a virus capable of injecting DNA into specific host cells) or transferred into  via crossing through aqueous  pores  the genetic material will make its way to the nucleus where another restriction enzyme will splice the ends of a DNA strand and with the complementary sequence for the specific plasmid (Cells must be heated to denature DNA first), then DNA ligase will seal up the gene into the strand . Once the plasmid has been inserted the cell will start expressing the specific gene. B)   splicing the insulin production gene is an example, this  is useful because  once in the cell  (usually bacteria) the genetic material will be passed through the generations of bacteria through mitosis and budding and can be used to produce insulin at a cheap price for diabetics.

48 LO 3.35: The student is able to create representation(s) that depict how cell-to-cell communication occurs by direct contact/from a distance through cell signaling SP 1.1: The student can use representations and models to communicate scientific phenomena and solve scientific problems. Explanation: Cell signaling occurs through the Signal Transduction Pathway. This begins with the signaling cell secreting a signaling molecule. This molecule (or ligand) reaches the targeted cell, and then binds to the cell. After this, transduction begins where a series of “relay” molecules pass the message to one another until the final one triggers the cellular response, where the cell can react accordingly. This Signal Transduction Pathway can be between cells that are directly next to one another, cells that are far apart, or even one cell that signals all of the cells surrounding it. The first receptor would be extracellular integral transmembrane proteins, which can be found within the plasma membrane. Intracellular membranes one the other hand have receptors within the actual cell itself. Molecules involved in transduction are growth factors, hormones, and neurotransmitters. These can range from signaling to cells directly adjacent, to a completely other side of the body. An example of a response would be gene activation. This is when organisms can adapt to specific situations since it allows for certain proteins to be expressed within the cell. FRQ: Using the signal transduction pathway and the figure provided, explain how a fight-or-flight response would be initiated in someone who has encountered an attack. MC Question: See next slide.

49 M.C. Question: Which of the following figures is the correct sequence of events for the Signal Transduction pathway? a. c. b. d.

50 ANSWER KEY FRQ: A fight or flight response would be triggered under an extraneous and dangerous situation upon the response of epinephrine. According to the figure it can clearly be seen that an environmental stimulus (in this case, the attack or whatever the cause may be), then a signal is sent to the brain that tells that body to respond by employing the use of epinephrine, so the body can fight back within the situation. M.C. Question: Which of the following figures is the correct sequence of events for the Signal Transduction pathway? c.

51 L.O. 3.43: The student is able to construct an explanation, based on scientific theories and models, about how the nervous systems detect external an internal signals, transmit and integrate information, and produce responses. SP 7.1: The student can connect phenomena and models across spatial and temporal scales. Explanation: Nervous systems have evolved in organisms billions of years ago. Their ability to sense and react with the changes in the environment it was increased the likelihood of survival and reproduction. An example is the use of chemotaxis to find food. Within these are nerve cells called neurons that send and receive information. Theses cells receive signals from other neurons through their dendrites which extend from the cell body. Another of the cell body’s extensions is a single axon from where the signal, often a neurotransmitter, is fired across the synapse to the dendrites of another neuron. The axon is covered by the myelin sheathe that insulates the signal. The Myelin sheathe is made up of individual Schwann cells separated by the Nodes of Ranvier. A neuron goes between an action potential (firing) and resting potential (non-firing) state. K⁺ and Na⁺ pumps affect the action and resting potential of the neuron (Figure 1.1) by actively transporting ions against their concentration gradients powered by ATP. Without this pump the gradients out disappear and so would the resting potential. When a neuron receives a signal from another neuron, Na channels open its gates to depolarize the membrane from resting potential. If the signal is enough to reach the neuron’s threshold the neuron reaches action potential and fires as Na⁺ ions flood into the cell. When the action potential falls the K⁺ gates open allowing K⁺ ions to flow down the concentration gradient and outside of the cell making it more negative. The cell then closes its K⁺ channels and the cell returns to the resting state. At the synapses the action potential depolarizes the membrane and opens voltage gated Ca²⁺ channels which causes the vesicles carrying the neurotransmitters to fuse to the presynaptic membrane. The neurotransmitter is released into the synaptic gap and binds to a ligand on the postsynaptic neuron’s membrane and releases receptors inside of it. These neurotransmitters are chemicals that make their way throughout the body, each signaling for a different function. When information from neurons reaches the cerebral cortex the signal is sent to the thalamus which redirects the signal to appropriate lobes in the brain (Figure 1.2). Some of these areas are primitive structures that we share with other mammals and reptiles such as the area of the parietal lobe that olfactory senses go. The cortex can then supply a motor behavior by sending signals from the brain and down the spinal column by motor neurons. Multiple Choice: A scientist examines a neuron that has open ligand gated ion channels and closed Na⁺ and K⁺ channels along with increased concentrations of Na and K inside the cell. This neuron _____? Is at resting potential Underdoing depolarization Falling from action potential About to activate a stretch-gated ion channel Just received a neurotransmitter Learning Log/ FRQ-style Question: Ouabain, also known as g-strophanthin, is a poison that inhibits the sodium-potassium pump in neurons. How would this inhibition affect the concentration gradients of a neuron before action potential? How would the change in concentration gradients effect the amount of neurotransmitters located in the synapses? Figure 1.2 Figure 1.1

52 Answer Key LO 3.43 Multiple Choice: A scientist examines a neuron that has open ligand gated ion channels and closed Na⁺ and K⁺ channels along with increased concentrations of Na⁺ and K⁺ inside the cell. This neuron _____? Is at resting potential Underdoing depolarization Falling from action potential About to activate a stretch-gated ion channel Just received a neurotransmitter Learning Log/ FRQ-style Question: Ouabain, also known as g-strophanthin, is a poison that inhibits the sodium-potassium pump in neurons. How would this inhibition affect the concentration gradients of a neuron before action potential? How would the change in concentration gradients effect the amount of neurotransmitters located in the synapses? There would be no concentration gradients for the K⁺ and Na⁺ molecules to go against. If the pump is disabled the K and Na ions would diffuse across the membrane creating an equilibrium that doesn’t allow the neuron to reach the resting potential before action potential is reached. If the pump is inhibited the neuron will stay in a state of continual depolarization due to Na staying inside of the cell. Since the nerve cell is in a state of depolarization the neuron can continue to fire if a stimulus reaches it’s threshold. This results in increased amounts of neurotransmitters in the synaptic gap.

53 LO 1.8: The student is able to make predictions about the effects of genetic drift, migration and artificial selection on the genetic makeup of a population. SP 6.4: The student can make claims and predictions about natural phenomena based on scientific theories and models. Explanation: Genetic drift is one of the basic mechanisms of evolution. In each generation, some individuals may leave behind a few more descendants (and genes) than other individuals because of chance. The genes of the next generation will be the genes of the "lucky" individuals, not necessarily the healthier or "better" individuals. This happens to ALL populations and there is no way to avoid it. Genetic drift affects the genetic makeup of the population but, unlike natural selection, through an entirely random process. The relationship between genetic drift and natural phenomena is also evident in natural selection, mutation, and migration. 1. In the 1980s, nearly 100,000 elephants were poached for their precious ivory tusks and in some regions 80% of those herds were killed off. However, other species living in the same area were not harmed as brutally as the elephants. This causes the genetic variation of the elephant population to dramatically decline unselectively which limits diversity and changes the species gene pool. This type of change is specifically known as… Genetic drift Artificial selection Temporal isolation The bottleneck effect The founder effect 2. Sickle cell anemia is a rare genetically inherited trait that is only expressed by the homozygous recessive phenotype (aa). Sickle cells have abnormal hemoglobin therefore developing the red blood cells into a sickle or crescent shape. This can occur among any group of humans living in the world. There is a ratio of 1 out of 25,000 people that are affected by the disease. Calculate the value of p and q. Round to the nearest hundredth. State the allele frequency for the three genotypes. Give THREE examples for each bullet listed below. The population must be very large Mating must be random No natural selection

54 State the allele frequency for the three genotypes.
1. In the 1980s, nearly 100,000 elephants were poached for their precious ivory tusks and in some regions 80% of those herds were killed off. However, other species living in the same area were not harmed as brutally as the elephants. This causes the genetic variation of the elephant population to dramatically decline unselectively which limits diversity and changes the species gene pool. This type of change is specifically known as… Genetic drift Artificial selection Temporal isolation The bottleneck effect The founder effect 2. Sickle cell anemia is a rare genetically inherited trait that is only expressed by the homozygous recessive phenotype (aa). Sickle cells have abnormal hemoglobin therefore developing the red blood cells into a sickle or crescent shape. This can occur among any group of humans living in the world. There is a ratio of 1 out of 25,000 people that are affected by the disease. Calculate the value of p and q. Round to the nearest hundredth. aa=q^2 =1/25000=  q=a=.006 p=1-q  p=  p=A=.994 State the allele frequency for the three genotypes. P^2+2pq+Q^2 (.994)^2+2(.994)(.006)+ (.00036)^2=1 =1 AA=98.8%, Aa=1.2%, aa=.036% Give THREE examples for each bullet listed below. The population must be very large A Large population ensures that chance will not disrupt genetic equilibrium. A small population may only have a few copies of a certain allele that exists. Example of this could be a bottleneck occurrence. Mating must be random Coral polyps will discharge their sperm into the sea and the currents will distribute them all over. The contact of the sperm with another coral’s egg is completely random. No natural selection No alleles are to be chosen over other alleles at equilibrium. If selection were to occur, the alleles would become more common. An example would be if a certain herbicide allows weeds to live in its habitat that has been sprayed with the herbicide, the allele for resistance may become more frequent in the population. URL- Date accessed- 4/27/15

55 LO 1.6: The student is able to use data from mathematical models based on the Hardy-Weinberg equilibrium to analyze genetic drift and effects of selection in the evolution of specific populations. SP 1.4: The student can use representations and models to analyze situations or solve problems qualitatively and quantitatively. SP 2.1: The student can justify the selection of a mathematical routine to solve problems. Learning Objective (LO) and Science Practice (SP) courtesy of Explanation: Hardy and Weinberg figured out that with random mating, no mutations, a large population, isolation, and no natural selection there would be no genetic drift, or evolution, within a population. They further advanced their theory with the mathematical equation p+q=1, in which they could predict genotype frequencies within a population; p being the dominant alleles and q being the number of recessive alleles in a population. The equation is then extended further so one could predict the percentage of a population that would be either homozygous dominant, heterozygous, or homozygous recessive. This equation is p² + 2pq + q² = 1.This allows one to see the frequencies of the alleles much like a punnett square shown in figure 1.1. This also allows for one to see the genetic drift of the frequencies over time for as long as one can see how many things exhibit the recessive allele (how many organisms are recessive / total organisms = q²) one can then use basic algebra to reach what q and p equal and once those are plugged into the equation the frequencies are gotten as shown in figure 1.2. Multiple Choice I. A population lives just off the coast of Florida. It is large and isolated. Every member is heterozygous and there are no mutations. II. A second population lives on the coast of Florida. There are no common species to mate with nearby and no reason to leave. It is a very large population. There is no preference to dominant or recessive phenotypes or mutations III. A third population lives on the coast of Florida. It is large and isolated with no mutations. There is a preference for those who have the dominant phenotype and those that are homozygous recessive tend to die before adulthood. Given these populations which one(s) would maintain equilibrium? I only II only III only I and II I and III FRQ Question (a) Given a population on a coast that is large, experiences no mutations, with random mating and no natural selection, but is not isolated what would be a possible event to cause this society to be in equilibrium? How would that event satisfy the requirements for Hardy-Weinberg equilibrium? (b) Also if the population size is 3000 and the organisms that show the recessive allele total to 700 what is the frequency of AA, Aa, and aa? Figure 1.2: Figure 1.1:

56 Answer Key LO 1.6 FRQ Question Multiple Choice
(a) Given a population on a coast that is large, experiences no mutations, with random mating and no natural selection, but is not isolated what would be a possible event to cause this society to be in equilibrium? How would that event satisfy the requirements for Hardy-Weinberg equilibrium? (b) Also if the population size is 3000 and the organisms that show the recessive allele total to 700 what is the frequency of AA, Aa, and aa? Show your work. a) With the idea of Hardy-Weinberg equilibrium being achieved only with a large population that has random mating, no mutations, and no natural selection along with being isolated the only part the given population is missing is the isolation. With a large geographical event such as a new river forming to make that section of the coast an isolated island the population, after time, would be isolated and therefore achieve equilibrium as it would still be large, have no mutations, random mating, and no natural selection along with the new addition of isolation. b) With the total population being 3000 and the organisms who have aa alleles being 700, q² or aa = 700/3000 which equals .23. The square root of q² and .23 means that q = .48. With p+q=1 then 1-.48=p. P = .52 p² = (.52)(.52) = .27 2pq = 2*.48*.52 = .50 AA or p² = .27 aa or q² = .23 Aa or 2pq = .50 Multiple Choice I. A population lives just off the coast of Florida. It is large and isolated. Every member is heterozygous and there are no mutations. II. A second population lives on the coast of Florida. There are no common species to mate with nearby and no reason to leave. It is a very large population. There is no preference to dominant or recessive phenotypes or mutations III. A third population lives on the coast of Florida. It is large and isolated with no mutations. There is a preference for those who have the dominant phenotype and those that are homozygous recessive tend to die before adulthood. Given these populations which one(s) would maintain equilibrium? I only II only III only I and II I and III The five requirements for equilibrium are no mutations, large population, isolation, random mating, and no natural selection. With those five requirements populations I and II meet them, but III lacks the requirement of random mating and no natural selection (dominant phenotype is prefered and recessive alleles tend to die before being able to reproduce). Therefore the correct answer is D.


Download ppt "4th Period Review Projects"

Similar presentations


Ads by Google